Normal & Abnormal Growth and Development/Child Health/Paediatrics Flashcards
Which one of the following is the most common cause of painful rectal bleeding in children?
A. Intussusception.
B. Gastroenteritis.
C. Anal fissure.
D. Meckel’s diverticulum.
E. Hemorrhoids.
Gastroenterology
Correct Answer Is C.
Anal fissure is the most common cause of painful rectal bleeding in childhood. It is usually caused by passage of hard stool associated with constipation. The common presentation is bright blood on the surface of stool, on the nappy or toilet paper.
(Option A) Intussusception is associated with abdominal pain and rectal bleeding, not usually with pain on defecation and not as common as anal fissure.
(Option B) Gastroenteritis and juvenile polyps are the most common causes of rectal bleeding in children aged 2 to 5 years, but these conditions are not associated with painful defecation.
(Option D) Meckel’s diverticulum can cause blood in the stool and abdominal pain but the pain is not related to defecation. Moreover, it is far less common than anal fissure.
(Option E) Hemorrhoids neither are associated with painful defecation, nor as common in childhood.
Concerned parents of a 7-year-old boy have brought him to your clinic after he passed a large bloody stool this morning. No pain or discomfort is reported. Which one of the following can be the most likely cause?
A. Meckel’s diverticulum.
B. Duodenal ulcer.
C. Colon polyp.
D. Diverticulitis of the colon.
E. Intussusception.
Gastroenterology
Correct Answer Is C.
A large bloody stool, especially if painless, in a 7-year-old child is very likely to be caused by a juvenile colonic polyp.
Juvenile polyps are benign hamartomas, occurring between the ages 2 and 8 years, with a peak at 3 to 4 years.
Patients usually present with painless rectal bleeding; however, a few children may have lower abdominal pain from traction of the polyp. More than 60% of patients have a palpable polyp on rectal examination.
(Option A) Meckel’s diverticulum can result in massive rectal bleeding, often associated with central abdominal pain.
However, the presentation is expected earlier in life and is also less common than juvenile polyps.
(Option B) Duodenal ulcer is most likely to present with melena or hematemesis rather than large rectal bleeding.
(Options D and E) Diverticular disease is not common among children of this age group, neither is intussusception.
Moreover, intussusception is often painful.
A 5-year-old boy is brought to your clinic with generalized petechial rash and bruises all over his body. He had an episode of upper respiratory tract infection (URTI) 3 weeks ago. On examination, he is otherwise healthy. His blood tests is only significant for a platelet count of 74,000/mm3. Red blood cells and white cells are within normal range.
Which one of the following is the most appropriate management option in this child?
A. Strict bed rest.
B. Intravenous immunoglobulin (IVIG).
C. Steroids.
D. Antibiotics.
E. Plasma exchange.
Haematology
Correct Answer Is A.
Isolated thrombocytopenia in an otherwise healthy child following an URTI is highly suggestive of immune (idiopathic) thrombocytopenic purpura (ITP). ITP in children is a benign disease of unknown etiology.
ITP can be acute (~90%) or chronic (~10%). Both forms have similar presentation. Twenty percent of those with acute ITP will go on to have the chronic form, defined as persistent thrombocytopenia more than 6 months. However, some authors have suggested that 12 months is a more appropriate cut-off for defining chronic ITP.
The diagnosis is made based on manifestations of thrombocytopenia in the absence of abnormal findings, in
particular no pallor, lymphadenopathy or hepatosplenomegaly. Confirmation relies on exclusion of other causes of thrombocytopenia, acute leukemia in particular.
The management of children with ITP is controversial. There are generally two different approaches: (1) watchful
waiting and (2) pharmacologic intervention. None has been proved superior to the other.
Most patients with a platelet count > 20 x 10
9/L and some of those with a platelet count < 20 x 10 9/L can be managed in outpatient setting with no specific treatment if the following criteria are met:
* The diagnosis is certain, and there is no pallor, hepatosplenomegaly or lymphadenopathy, and there is only
isolated thrombocytopenia without anemia, leukopenia or blood film changes on laboratory studies
* There is no active bleeding other than bruising and petechiae in isolation. There should be no mucosal,
gastrointestinal or renal tract bleeding
* The child is otherwise well
* Social circumstances allow confidence about the degree of parental supervision and relative safety of the home environment, particularly for younger children.
* Parental reassurance and education in the emergency department can be provided.
* Follow up is guaranteed within a few days.
(Option B) Intravenous immunoglobulin (IVIG) is not routinely used and is reserved for patients with severe lifethreatening hemorrhage.
(Option C) This child has platelet count of >20,000/mm3, has no hemorrhage other than bruises and petechial rash and is otherwise healthy. He does not need pharmacological treatment. However, if pharmacological intervention is
planned, oral steroids are first-line.
(Option D) Antibiotics have no role in management of ITP.
(Option E) Plasma exchange is not an option for treatment of ITP
A mother brings her 7-year-old son 6 days after eruption of a generalized rash that, based on the history and clinical
findings, is found to be measles. She asks if her son is required to be excluded from school. She also mentions that according to the class teacher, all students in the class have complete vaccination against measles.
Which one of the following would be the most appropriate advice regarding exclusion from school for this boy?
A. No exclusion is required.
B. Exclusion for 4 days.
C. Exclusion until the rash completely disappears.
D. Exclusion is required only if there is an immunocompromised child in the class room.
E. Exclusion for 6 days
Infectious Disease, Vaccination, Exclusion
Correct Answer Is A.
Measles is highly infective and all suspected, probable and confirmed cases should be excluded from work, school, and early childhood education and care services. Patients with established measles should be excluded until 4 days after the onset of rash. This child has had eruption of his rash 7 days ago; therefore, he is not infective anymore and exclusion is not required.
TOPIC REVIEW
Since measles is highly infective, it is very important that persons who has been in contact with a person with measles (index case) are traced and excluded from contacts to others accordingly. It is even more crucial in childcare setting. The following are recommendations regarding exclusion of contacts of known case of measles in
childcare setting:
* Immunized contacts of a child with measles do not require exclusion.
* Non-immunized children, who have been in contact with a child with measles should be excluded from school until 14 days after the day the infected child developed the rash, unless they are immunized with MMR within
72 hours of contact with the infected person.
Example: John develops a rash 3 days ago. There are four other children in his class: Alan, Josh, Mary, and Ana. These
children are all 7 years old. Ana contracted measles at the age of 3, and Josh has had all his childhood vaccines to date.
Alan and Mary have not received any vaccines because their parents are against immunization. So exclusion from
school for these children would be:
- John (the index case): until tomorrow which is 4 days after the onset of the rash (3 days ago).
- Ana (contact): no exclusion is required because she had measles and is immune now.
- Josh (contact): no exclusion is required because he is up-to-date with his vaccination.
- Alan (contact): he is not immune so he should be excluded from school for 11 days from today because John developed the rash 3 days ago.
- Mary (contact): she is not immune so she should be excluded from school for 11 days from today because John developed the rash 3 days ago.
A 5-year-old boy is presented to your GP clinic by his parents with intermittent abdominal pain and vomiting usually lasting for 12 hours. The last painful episode was felt in the left flank. Physical examination is quite inconclusive.
You order a urine analysis and culture, the results of which are negative. Which one of the following investigations is most likely to reveal the underlying cause of this presentation?
A. Plain abdominal films.
B. Small intestine barium meal and follow-through.
C. Serum creatinine and BUN.
D. Serum electrolytes.
E. Ultrasound scan during a painful episode.
Urology
Correct Answer Is E.
Intermittent abdominal pain in children has a myriad of causes. In approximately 40%, no apparent cause is found after extensive investigations. Such pains are referred to as functional abdominal pain.
In older children, periodic abdominal pain with vomiting is a common symptom. These symptoms are probably caused by intermittent kinking of the UPJ. If obstruction does not resolve, severe hydronephrosis and pelvic distention can follow that presents with flank pain or even tenderness. Although rarely seen in developed countries, massive dilation of the renal pelvis and kidney can fill the entire flank and abdomen.
In younger adults, the typical presentation is episodic flank pain following diuresis.
Ultrasonography is the initial diagnostic choice for most cases of hydronephrosis and UPJ obstruction. Since the condition is present intermittently, sonography during a painful attack is most likely to delineate the diagnosis.
(Option A and B) Abdominal X-ray films and barium and follow through are indicated if intussusception is the suspected diagnosis. Intussusception can present with intermittent abdominal pain; however, most cases present
within the first 2 years of life (2 months -2 years). This child is 5 years old. Moreover, flank pain is unlikely to be a presentation of intussusception.
(Options C and D) Electrolytes, BUN, and creatinine might be indicated for evaluation of kidney function but they are
of little, if any, diagnostic value.
A 4-year-old is boy is brought to your attention with complaints of intermittent abdominal pain. Each episode lasted for a maximum of 2 to 3 hours before it completely subsided. He has been quite asymptomatic in between the episodes. This time, however, the pain has lasted for 12 hours. On examination, he is afebrile but a mass is palpated in the right upper quadrant (RUQ). The mass is not tender. Which one of the following could be the most likely
diagnosis?
A. Hepatoblastoma.
B. Neuroblastoma.
C. Wilms tumor.
D. Pyelonephritis.
E. Pancreatic tumor.
Correct Answer Is C.
The scenario is consistent with Wilms tumor (nephroblastoma) as the most likely diagnosis. Wilms tumor is the most common intraabdominal tumor of childhood that often is diagnosed at 2 to 3 years of age.
Most cases are sporadic, and only a few percent have a family history. Wilms tumor suppressor gene, WT1, is located on chromosome 11 and regulates normal kidney development. In approximately 20% of cases with Wilms tumor, there is a mutation of this gene.
Most children with Wilms tumor presents with abdominal mass or swelling without other signs and symptoms. Other symptoms, if present, can include abdominal pain (between 25-40% according to different studies), hematuria (12-25%), fever, and hypertension (25%). Hypertension is the result of the tumor compressing the renal artery, renal hypoperfusion, and activation of renin-angiotensin-aldosterone system.
The characteristic finding on examination is a smooth firm non-tender palpable abdominal mass that usually does not cross the midline.
(Option A) Hepatoblastoma is rare hepatic malignancy in children. Patients with hepatoblastoma are usually asymptomatic. The disease is advanced at diagnosis in approximately 40% of patients, and 20% have pulmonary metastases. Children with advanced disease may have anorexia. Severe osteopenia is present in most patients and regresses with resection of the tumor; osteopenia is often asymptomatic. Rarely, patients in whom the tumor has
ruptured present with symptoms consistent with acute abdomen. Occasionally, patients present with severe anemia resulting from tumor rupture and hemorrhage.
(Option B) Neuroblastoma, which is almost exclusively a disease of children, is the third most common childhood cancer after leukemia and brain tumors, and is the most common solid extracranial tumor in children. Neuroblastoma can also present with an abdominal mass, but the condition is expected to be diagnosed earlier compared to Wilms tumor, usually before the age of 2 years. Two-thirds (~65%) of neuroblastomas arise within the abdomen, of these 2/3 originate from adrenal glands. The abdominal mass seen in neuroblastoma is hard, irregular and non-tender and can extend beyond the midline. Other symptoms may include loss of appetite and weight loss, malaise, protrusion of one or both eyes. Other symptoms may be caused by compression effect the of tumor or metastases on the adjacent structures.
(Option D) Pyelonephritis can be associated with costovertebral angle tenderness but not a palpable mass. Fever is often a feature. Most importantly, it does not manifest as long standing intermittent abdominal pain with full resolution.
(Option E) Pancreatic tumors are extremely rare in children.
A 6-year-old girl is brought to your practice by her parents with a 4-day history of bilateral knee and ankle pain. On examination, she has a temperature of 37.6 C. Her joints are tender but there is no swelling, inflammation, or synovial thickening. The full range of motion of the joints is preserved. Palpable purpuric rashes are noted on her legs and buttocks with some ecchymoses around her ankles. Full blood count, blood urea, electrolytes and liver function tests are normal. ESR is 10mm/hr. Which one of the following is a known complication of this girl’s condition?
A. Pleurisy.
B. Fasciitis.
C. Nephritis.
D. Thrombocytopenia.
E. Diabetes mellitus.
Correct Answer Is C.
The clinical picture is highly suggestive of Henӧch-Schonlein purpura (HSP).
HSP, also known as anaphylactoid purpura, is an acute immune-mediated vasculitis of unknown cause. It is characterized by arthralgia (not arthritis), and non-thrombocytopenic purpura (the purpura is vascular due to leukoclastic activity) with typical distribution over legs and buttock.
Other clinical manifestations HSP include colicky abdominal pain, melena, swelling of the joints especially the ankles, and nephritis. The arthralgia and abdominal pain may persist for 2 to 4 weeks.
Nephritis of HSP, which is histologically the same as the IgA nephropathy, may occur in a small percentage of patients. For this reason, all patients with HSP should have a urinalysis. If the patient is found to have hematuria, long-term follow-up is required.
A 4-year-old child is brought to your practice with complaint of intermittent right upper quadrant (RUQ) and right flank pain for 2 years. These painful episodes frequently have been associated with vomiting and lasted for 3 hours. He has been well in between the episodes. This time, the pain has lasted for 12 hours. On examination, a mass is palpated in the RUQ. The mass is tender to palpation. Which one of the following is the most likely diagnosis?
A. Wilms tumor.
B. Neuroblastoma.
C. Ureteropelvic junction obstruction.
D. Vesicoureteral reflux (VUR).
E. Pyelonephritis.
Correct Answer Is C.
Intermittent abdominal pain in children is a frequent reason of seeking medical attention by concerned parents. The causes of intermittent abdominal pain are diverse, ranging from benign conditions such as functional abdominal pain to grave diseases such as childhood malignancies.
As usual, a thorough history and physical examination is the cornerstone of approach to such children. Associated symptoms can also help narrow down the differential diagnoses.
Of the options, ureteropelvic junction (UPJ) obstruction is the only likely diagnosis justifying the clinical scenario. UPJ obstruction is by far the most common cause of pediatric hydronephrosis, occurring in 1 per 1000-2000
newborns. Widespread use of antenatal ultrasonography and the advent of modern imaging techniques have resulted in earlier and more common diagnosis of the condition.
In older children, periodic abdominal pain with vomiting is a common symptom. These symptoms are probably caused by intermittent kinking of the UPJ. If obstruction does not resolve, severe hydronephrosis and pelvic distention can follow, presenting with flank pain or even tenderness. Although rarely seen in developed countries, massive dilation of the renal pelvis and kidney can fill the entire flank and abdomen.
In younger adults the typical presentation is episodic flank pain following diuresis.
(Option A) Wilms tumor is the most common intra-abdominal malignant tumor of childhood that often presents with
a smooth firm abdominal mass that usually do not cross the midline. Abdominal pain, either constant and vague or intermittent, is another finding. Hematuria and hypertension are other possible manifestations. Unlike the mass in the scenario, the abdominal mass found in Wilms tumor is non-tender, making Wilms tumor a less likely diagnosis.
(Option B) Neuroblastoma is the most common extracranial malignant tumor of childhood. The initial presentation is
most commonly within the first 2 years of life. The tumor is intraabdominal in two-thirds of cases. Of these cases, two-thirds originating from adrenal glands. It can also present with a palpable abdominal mass that is non-tender.
However, the age of the child and the tenderness of the mass are against neuroblastoma as the diagnosis.
(Options D and E) Based on physical finding of a palpable mass, pyelonephritis or vesicoureteral reflux are very unlikely to be the diagnosis as, these do not cause a palpable abdominal mass.
Mother of a 6-month-old boy brings him for evaluation after she felt a hard mass in the left hypochondrium while she was bathing him. The mass is non-tender. Which one of the following could be the most likely diagnosis?
A. Wilms tumor.
B. Neuroblastoma.
C. Polycystic kidney disease.
D. Ureteropelvic junction obstruction.
E. Hepatoblastoma.
Correct Answer Is B.
In approach to a child with an abdominal mass two possibilities should always be considered: (1) Wilms tumor
(nephroblastoma) and (2) neuroblastoma.
Wilms tumor is the most common intra-abdominal malignant tumor of childhood that often presents with a smooth firm abdominal mass that usually do not cross the midline. Abdominal pain either constant and vague or intermittent is another finding. Hematuria and hypertension are other possible manifestations. The median age of diagnosis is 3.5 years. Although not impossible, it is less likely to present within the first year of life.
Neuroblastoma is the most common extracranial malignant tumor of childhood with the initial presentation most commonly being within the first 2 years of life. The tumor is intraabdominal in two-thirds of cases, of these twothirds originating from adrenal glands. It can also present with a palpable abdominal mass that is non-tender.
Given the age, neuroblastoma is more likely than Wilms tumor to be diagnosis. Wilms tumor and less frequently neuroblastoma are often found when the child is being bathed or hugged.
(Option A) Given the age of the baby, Wilms tumor is a less likely dagnosis compared to neuroblastoma.
(Option C) An enlarged cystic kidney may be palpated as an abdominal mass in patients with polycystic kidney disease, but the mass is not hard.
(Option D) Ureteropelvic junction obstruction is not associated with a hard mass. In fact, if a mass is present, it is the significantly dilated kidney which is not hard, but can be tender.
(Option E) Hepatoblastoma is a rare hepatic malignancy in children. It does not give rise to a hard mass on the left side.
A previously healthy 18-month boy is brought to the Emergency Department with wheezing and cough starting 24 hours ago. On examination, he has a respiratory rate of 35 breaths per minute. Bilateral wheeze is evident on chest auscultation, as well as mildly diminished breath sounds of the left hemithorax. There is family history of asthma, as well as allergy in history. Which one of the following is the most appropriate next step in management of this child?
A. Nebulized salbutamol 4-hourly.
B. Amoxicillin.
C. Admission and administration of oxygen.
D. Chest physiotherapy.
E. Chest X-ray (CXR).
C. Admission and administration of oxygen.
Wheezing in children have a multitude of causes. The age of the child, physical findings, and the pattern and associations can help narrow down the differential diagnoses.
For instance, wheezing associated with feeding points towards reflux disease as the most likely cause, while wheeze and cough may suggest asthma. Positional wheeze, for example when the child is supine, is suggestive of laryngomalacia, whereas, wheeze in the presence of fever may be due to infections. Sudden onset unilateral
wheezing is a strong clue to foreign body aspiration.
The age of onset is another important clue to the diagnosis. Early onset wheeze can reflect a congenital problem.
Bronchiolitis is a very common cause of wheezing in children younger than 12 months, and asthma is the most
common cause of episodic wheezing in children younger than 5 years. Despite this, a definite diagnosis of asthma is avoided at this age group and other terms such as wheeze syndrom, viral wheeze, etc. are used instead.
In this child, wheezing and cough is of sudden onset, suggesting aspiration; however, bilaterality of the wheeze on exam makes this diagnosis very unlikely. The presence of cough and the strong family history of asthma and allergy put asthma at the top of the differential diagnoses list. This child though has also diminished breath sounds of the right lung that may suggest collection either in the lung or pleural space. This child needs to be admitted and through investigation carried out; however, making sure of adequate oxygenation regardless of the diagnosis comes first.
(Option A) Based on the provisional diagnosis of asthma, a trial of salbutamol is required both diagnostically and therapeutically because adequate response to bronchodilators is in favor of a diagnosis of asthma. This should be performed after oxygenation has started. Salbutamol is given by nebulizer in this age group. An adequate response to therapy almost clinches the diagnosis with high certainty.
(Option B) Antibiotics such as amoxicillin may be necessary later on if investigations suggest bacterial infections such as pneumonia.
(Option D) Chest physiotherapy is of little, if any, value at this point.
(Option E) CXR is indicated in children who present with unexplained wheezing that is unresponsive to bronchodilators or with recurrent wheezing. It is not indicated in this child who has presented with first-time wheeze that is very likely to be due to asthma. It, however, might be indicated if the wheeze is proved to be due to other causes than asthma, e.g., infections, foreign body, etc.
Which one of the following is the most common cause of failure to breastfeed?
A. Exhaustion of the mother.
B. Reduced frequency of breastfeeding.
C. A reduction in the time the baby feeds at each session.
D. Maternal dehydration.
E. Nipple problems.
Correct Answer Is B.
The benefits of breastfeeding for infants and mothers are well proven. Recommendations to breastfeed exclusively for 6 months (EBF6) have been widely adopted by relevant organizations in Australia.
Advantages of breastfeeding for the child include increased resistance to diseases, lower rates of diseases such as gastroenteritis, recurrent otitis media, and pneumonia. Additionally, there are lower rates of sudden infant death syndrome (SIDS), colitis, hypertension, obesity, hyperlipidemia, atopic disease and diabetes, and a higher IQ.
For the mother, there is a delay in ovulation, decreased risks of breast and ovarian cancer and the bonding effects of breastfeeding.
For many women, difficulties in breastfeeding result in early termination of breastfeeding before the recommended period of 6 months.
By far, inadequate milk intake or the perception of inadequate milk production is the most common reason for early
termination of breastfeeding. Inadequate milk intake may be due to failure of the infant to extract milk or insufficient milk production.
Inappropriate early feeding routines are the most common cause of insufficient milk intake. They include infrequent feeding, poor latch-on, maternal-infant separation, and the use of supplemental formula.
Other less common causes include oral-motor or neurologic abnormalities of the baby and poor emptying of the breast.
Of the given options, reduced frequency of breastfeeding is the most common cause to decreased milk production.
With infrequent feeds, milk production reduces resulting in both inadequate milk production by the mother and intake
by the baby.
(Option A) Maternal exhaustion can contribute to decreased frequency of breastfeeding and eventually failure of
lactation.
(Option C) Reduced time of latching during each feed does not have such significant negative impact on milk
production as has reduced frequency of feeds.
(Option D) Maternal dehydration can be associated with decreased milk production but is not as common a cause as
is infrequent feeding.
(Option E) Nipple problems can prevent from appropriate and adequate breastfeeding; however, infrequent feeds
remain the most common cause of failure to breastfeed.
A 10-year-old girl is being assessed in your clinic with complaints of labial fusion. She has the past history of vulvovaginitis which has been successfully treated. The child reports no voiding difficulties. Which one of the following is the most appropriate next step in management?
A. Reassurance.
B. Surgical separation of adhesions.
C. Encourage good hygiene.
D. Antibiotics.
E. Manual separation of adhesions.
Correct Answer Is A.
Labial adhesion (labial agglutination) occurs when the medial edges of the labia minora becomes adherent, often in the background history of vulvovaginitis. This is probably due to combination of thin vaginal mucosa (the normal prepubescent state) and minor irritation (such as in vulvovaginitis).
This is a normal variant and will resolve spontaneously in late childhood. Provided that the child is able to void easily, no treatment other than reassurance is required.
(Option B) Surgical separation of the adhesions, followed by application of Vaseline and/or estrogen cream can be considered for children with urinary symptoms. This method, however, is not generally recommended because of the high recurrence rates.
(Option C) Encouraging good hygiene, althoug a good advice, is not effective in treatment of the condition.
(Option D) Antibitoics have no role in management of labial adhesions.
(Option E) Manual separation of adhesions can be distressing for the child and is associated with a high risk of recurrence and not recommended.
A 9-year-old boy is brought to your attention by his parents because of poor exercise tolerance. He is a member of the school basketball team but recently, it has been hard for him to play because he becomes short of breath and tired soon after he starts playing. On examination, his height is at 97 percentile and weight at 3 percentile. Which one of the following is the most important step in management?
A. Karyotyping.
B. Bone scan.
C. Cardiac ultrasound.
D. Growth hormone test.
E. CT scan.
Correct Answer Is C.
Marfan syndrome (MFS) should always be considered as a probable diagnosis in children with tall stature and low
weight, MFS is an autosomal dominant connective tissue disorder with involvement of the cardiovascular, skeletal and
ocular systems, as well as the skin, lungs and dura. The prevalence is at least 1 in 5000. In 90-93% of cases, MFS is caused by mutations in FBN1. In a minority, MFS is caused by mutations in a second gene called TGFBR2. MFS caused by this mutation is termed MFS2. Based on current statistics, MFS2 accounts for up to 10% of cases with MFS.
Cardinal manifestations include aortic aneurysm and dissection, ocular lens dislocation and long bone overgrowth.
Cardiac disease is the leading cause of morbidity and mortality in patients with MFS. When the syndrome is suspected based on clinical grounds, an echocardiogram (cardiac ultrasound scan) should follow as the most
appropriate next step in management for evaluation of aortic root dilation (the most important) and mitral valve prolapse – conditions frequently seen in patients with MFS. The former is more common and significant clinically.
Serial echocardiographic surveillance is indicated for all affected individuals. Frequency should be tailored to each
individual by their cardiologist. Based on current evidence, the use of beta blockers remains the first-line treatment (except where contra-indicated e.g., in asthmatic patients) in aortic dilatation in MFS even in young children if a diagnosis of MFS is clear, or if there is a known FBN1 mutation in a young child without clinical features of MFS but with affected first degree relative(s) with a known mutation and aortic root dilatation. If beta blockers are ineffective or contraindicated, verapamil or ACE inhibitors would be the appropriate second-line treatment options.
A 14-year-old boy has clinically apparent Marfan syndrome. Which of the following cardiac murmurs would you expect to hear on cardiac auscultation?
A. Midsystolic ejection murmur at the second right intercostal space.
B. Decrescendo high-pitched diastolic murmur at the left sternal edge.
C. Low-pitched rumbling diastolic murmur at the apex.
D. Pansystolic murmur at the left sternal edge with no radiation.
E. Continuous machinery murmur at the second left intercostal space.
Cardiology, Murmurs
Correct Answer Is B.
Aortic root disease is the most common cardiac manifestations in Marfan syndrome (MFS). This results in aneurysmal dilation, aortic regurgitation and aortic dissection. Such conditions are the most common causes of mortality and morbidity in patients with MFS.
Dilation of the aorta is found in approximately 50% of children and 60-80% of adults with MFS. Aortic regurgitation (AR) is often present. On cardiac exam, murmurs caused by chronic AR are most likely to be heard.
The murmur of AR is a decrescendo murmur occurring during diastole usually as a high-pitched sound that is loudest at the left sternal border. The duration of the murmur correlates with the severity of AR; loudness of the
murmur does not.
Other possible murmurs in patients with MFS and AR include:
Austin-Flint murmur - During diastole, blood jet backs through the incompetent aortic valve and strikes anterior leaflet of the mitral valve and results in premature disclosure of this valve. This produces a middiastolic rumbling murmur best heard over the apex. Austin-Flint murmur is heard in patients with severe AR.
A functional systolic flow murmur may also be present due to increased stroke volume.
NOTE - Patients with MFS may also have mitral valve prolapse. The classic auscultation finding in mitral valve prolapse is a mid-to-late systolic click, which is present due to the leaflets prolapsing into the left atrium resulting in tensing of the mitral valve apparatus. This click may or may not be followed by a high-pitched, mid-to-late systolic
murmur at the cardiac apex.
(Option A) Midsystolic ejection murmur at the second right intercostal space is the characteristic finding in aortic
stenosis and hypertrophic obstructive cardiomyopathy (HOCM).
(Option C) A low-pitched rumbling mid-diastolic murmur over the apex (Austin-Flint murmur) is also possible in patients with MFS with severe AR. this is, however, a less common finding.
(Option D) Pansystolic murmur at the left sternal edge with no radiation is characteristic for ventricular septal defect.
(Option E) A continuous machinery murmur at the second left intercostal space is characteristic of patent ductus arteriosus (PDA).
A 5-week-old female infant is brought to your office by her parents because of ‘feeding problems’. Parents tell you
that their child has been vomiting after many of her feeds for the past week or so. They mention that this is a new problem, and she was previously quite healthy. For the first time, the baby’s vomit was projectile and forceful last night. On examination, a mass is palpated in the epigastric area. Which one of the following, can the most likely
metabolic profile in this infant?
A. Increased pH, increased K, increased Cl, increased pCO2, and increased HCO3.
B. Decreased pH, decreased K, decreased Cl, increased pCO2, and decreased HCO3.
C. Increased pH, increased K, decreased Cl, increased pCO2, and decreased HCO3.
D. Increased pH, decreased K, decreased Cl, increased pCO2, and increased HCO3.
E. Increased pH, decreased K, decreased Cl, decreased pCO2, and decreased HCO3
Correct Answer Is D.
The scenario describes a classic presentation of infantile hypertrophic pyloric stenosis (IHPS). IHPS is more common in males than females (4:1 to 6:1).
IHPS typically presents with immediate post-prandial non-bilious projectile vomiting in a 3- to 6-week old baby. After vomit, the baby demands to be re-fed immediately (hungry vomiter).
On physical examination, the baby is dehydrated with a palpable ‘olive-like’ mass at lateral edge of the rectus abdominus muscle in the right upper quadrant of the abdomen. The mass is most easily felt immediately after
vomiting.
Gastric fluid is rich in hydrochloric acid and potassium. Loss of gastric fluid by emesis results is hypochloremic metabolic alkalosis. Initially the potassium is normal, but as emesis persists hypokalemia develops. Metabolic alkalosis is associated with increased HCO3. To compensate the metabolic alkalosis, respiratory acidosis occurs that is associated with increased arterial pressure of CO2 (pCO2).
Given these, the expected metabolic picture in a patient with prolonged emesis will be increased PH (metabolic alkalosis), decreased serum potassium and chloride, increased HCO3 and increased PCO2.
Kyle, 5 months old, is brought to your practice by his mother for evaluation. He has had a runny nose and a fever or 40°C for the past 48 hours, but otherwise quite active and healthy. Today, Kyle’s temperature dropped to normal but he developed a maculopapular rash on his trunk. He is still in good health. Which one of the following is the most likely diagnosis?
A. Measles.
B. Chicken pox.
C. Erythema infectiosum.
D. Rubella.
E. Roseola infantum.
Correct Answer Is E.
The scenario is typical for roseola infantum as the diagnosis. Roseola infantum, also known as exanthem subitum, sixth disease, and three-day fever, is a clinical syndrome most frequently caused by human herpes virus 6. Other causes include human herpes virus 7 (HHV-7), enteroviruses (coxackievirus A and B, echovirus), adenovirus, and parainfluenza virus type 1.
It is a clinical syndrome of infants (mostly 6-18 months) characterized by 3 to 5 days of high fever of commonly up to 40°C (or even higher). The fever then resolves abruptly, followed by development of a rash.
.
Typical rash of roseola infantum is a macular or maculopapular rash, starting on the neck and trunk and spreading to the face and extremities. Uncommonly, the rash is vesicular. It is unusual for the rash to be pruritic. The rash typically persists for 1 to 2 days, but occasionally may come and go within 2 to 4 hours. In children receiving antibiotics due to the high fever, the onset of rash can be confused with drug allergy. Diagnosis is almost always clinical. The disease has a benign course and resolves spontaneously. Febrile convulsions may occur.
Lymphadenopathy and erythematous tympanic membrane are common. During the febrile phase, the disease can be confused with otitis media. The combination of high fever and bulging fontanelle occurs in as many as 26% of infants and can be mistaken for meningitis.
Which one of the following is suggestive of hypertrophic pyloric stenosis?
A. Vomiting after 1 hour of feeding.
B. Vomiting immediately after feeding.
C. Vomiting after few minutes of feeding.
D. Bilious vomiting.
E. Vomiting after 2 hours of feeding.
Correct Answer Is C.
Hypertrophic pyloric stenosis (HPS) is caused by progressive thickening of the circular muscle of the pylorus. This leads to gastric outlet narrowing. The condition usually presents between 2-4 and 6-9 weeks of age.
Predisposing factors to HPS are:
* Male gender
* First born
* Caucasian
* Parental history of HPS (higher if mother affected)
HPS typically presets initially with non-bloody, nonbilious vomiting at 2-9 (typically 4-8) weeks of age. Vomiting may initially be infrequent, but over several days it becomes more predictable, occurring at nearly every feeding.
The vomiting occurs within the first hour after meals, but the typical time is within the first few minutes (up to 30 minutes). Vomiting intensity increases until pathognomonic projectile vomiting develops.
Slight hematemesis of either bright-red flecks or a coffee-ground appearance is sometimes observed. Patients are usually not ill-looking or febrile. The baby in the early stage of the disease remains hungry and sucks vigorously after episodes of vomiting (hungry vomiter).
Prolonged delay in diagnosis can lead to dehydration, poor weight gain, malnutrition, metabolic alterations, and lethargy. Parents often report trying several different baby formulas because they (or their physicians) assume vomiting is due to intolerance.
A 13-year-old boy presents to his general practitioner, accompanied by his mother, complaining of poorly localized pain in his knee for the past 6 weeks. His mother has observed a limp for the same period of time. He walks with his foot externally rotated. Examination of the knee is normal. Which one of the following is the most likely diagnosis?
A. Osteochondritis dissecans of the knee.
B. Perthes disease of the hip.
C. Undiagnosed congenital dislocation of the hip.
D. Slipped capital femoral epiphysis (SCFE).
E. Osgood-Schlatter osteochondritis of the knee.
Correct Answer Is D.
The clinical features described, as well as the age of the patient suggests slipped capital femoral epiphysis (SCFE) as the most likely diagnosis.
SCFE is more commonly seen in adolescents of 10 to 15 years of age. The classic case would be an oversized prepubertal boy. The condition is bilateral in 20% of cases.
SCFE presents with the following:
* Limp and irritability of hip on movement
* Knee pain – referred from the affected hip
* On flexion of the hip, it rotates externally.
* Hip is often in external rotation on walking.
* Most movements restricted, especially internal rotation.
The first symptom is hip stiffness that subsides with rest. Later on, limping and hip pain radiating down the anteromedial thigh to knee follows. Early hip examination neither detects pain nor movement limitation. In more advanced stages, hip movements become painful and there is decreased flexion, abduction and internal rotation.
The affected leg is externally rotated on walking.
The most significant aspect of the SCFE is the great number of patients who develop avascular necrosis of the femoral head despite expert treatment. Therefore, diagnosis of the condition before major slipping occurs is important. This necessitates early investigation and referral.
Any adolescent with a limp or knee pain should have X-rays (AP and frog view) of both hips. Otherwise, this important condition will be overlooked.
The most important management principles include:
* Cease weight-bearing and refer urgently.
* If acute slip, gentle reduction via traction is better than manipulation for prevention of later avascular necrosis.
* Once reduced, pinning is performed.
(Option A) Osteochondritis dissecans is characterized by separation of an osteochondral fragment from the articular
surface. The underlying bone from which the fragment separates has normal vascularity. Knee pain and swelling are clinical features of the disease. Hip is not affected.
(Option B)** Legg-Calvé-Perthes disease**, also called Perthes disease, is a temporary condition in the hip joint characterized by decreased blood supply to the femoral head and consequent avascular necrosis. As a result, femoral head collapses and the area becomes inflamed and irritated. Legg-Calvé-Perthes disease causes the hip joint to become painful and stiff. Affected children are usually between 4 and 10 years old, physically active and small for their age.
(Option C) It is very unlikely for congenital dislocation of the hip to remain asymptomatic until this age. The recent onset of the problem makes this diagnosis unlikely.
(Option E) Osgood-Schlatter disease is seen in preadolescent children and is characterized by pain localized to the tibial tubercle and occasionally the patellar tendon. The pathophysiology of the disease is by repetitive traction effect of patellar tendon on an immature tibial tubercle. There is often tenderness over the tibial tubercle on examination. Hip movements are not painful or restricted.
A 4-year-old boy is brought to your practice by his parents because of what they think to be vertigo. The boy suddenly starts feeling that the world is spinning around him. Each episodes lasts for about one to two minutes, while the child is fully conscious. This has happened once a month for the past three months. His past medical history isunremarkable. He has had no headache, nausea, or vomiting before, during, or after the attacks. Physical examination including ear exam is normal. Which one of the following is the most appropriate next step in management?
A. Reassure the parents as this is very likely to be benign paroxysmal positional vertigo (BPPV).
B. EEG and CT scan of the head.
C. Audiology and ENT referral.
D. MRI of the head.
E. Psychiatric assessment.
Correct Answer Is B.
Although dizziness and vertigo can be confused by patients, the spinning sensation described by the child is most likely true vertigo rather than dizziness. Unlike in adults, vertigo in children often has a sinister cause such as brain tumors (e.g., medulloblastoma) or temporal lobe epilepsy.
In every child with vertigo, epilepsy and CNS tumors should be excluded as the most appropriate next step in management, and early referral for specialist review must be considered.
In addition to epilepsy and CNS tumors, the following can also cause vertigo in children:
* CNS infections
* Trauma, especially to the temporal area
* Middle ear infections
* Prescription drugs
* Alcohol and other illicit drugs
* BPPV (rare)
* Labyrinthitis
(Option A) BPPV in children is considered a migraine variant of childhood and presents quite similar to this scenario.
However, BPPV is rare in children and is a diagnosis of exclusion, only made once other causes have been excluded. While CNS tumors are more common and serious diagnoses, reassuring the parents based on a hasty diagnosis of BPPV is inappropriate.
NOTE – BPPV in childhood often precedes migraine in adulthood.
(Option C) Generally, neurology referral (not ENT referral) is an appropriate and safe option to select because CNS tumors and epilepsy are the major differential diagnosis in such presentation in a child. Audiology assessment and ENT referral are appropriate once CNS-related causes have been safely excluded.
(Option D) MRI may be considered later if a CNS tumor is suspected during initial evaluation with CT scan.
(Option E) This child does not seem to have a psychiatric problem, and psychiatric assessment is not an appropriate
initial management option.
A 3-year-old girl is brought to your practice by her mother with complaint of multiple painful small oral ulcers. The child refuses to eat because of the pain. On examination, the child is found to have a temperature of 38°C. Similar ulcers are noted on his hand and feet. Which one of the following is the causative organism?
A. Herpes simplex virus (HSV).
B. Group B streptococcus (GBS).
C. Coxsackie A virus.
D. Adenovirus.
E. Coronavirus.
C. Coxsackie A Virus
The clinical picture of ulcers in mouth and on hands and feet is highly suggestive of ‘hand, foot and mouth (HFM) disease’. This often self-limiting mild viral infection is causes by Coxsackie A virus. The disease has an incubation period of 3 to 5 days. The illness begins with a prodromal syndrome of mild fever, headache and malaise, followed by
appearance of the rash after 1 to 2 days. The rash starts as an erythematous macular rash, progressing to gray vesicles with surrounding erythema. These vesicles form shallow ulcers on buccal mucosa, gums and tongue.
Hands and feet, especially the lateral borders, are affected as well. It is not uncommon to see the rash on buttocks and genitalia as well.
The rash resolves in 3 to 5 days with no scarring. The child is infectious until blisters disappear, but the virus can be secreted into saliva and feces.
A 10-year-old boy is referred to your clinic from his school for assessment of probable hearing deficit because he has difficulty in hearing resulting in dropped school performance. You carry out a Rinne test that is positive in both ears. A Weber test shows no lateralization; however, he has symmetrical decreased hearing threshold. Which one of
the following options could be the most likely diagnosis?
A. Presbycusis.
B. Otosclerosis.
C. Congenital sensorineural deafness.
D. Chronic perforation of the tympanic membrane.
E. Chronic secretory otitis media with effusion.
C. Congenital sensorineural deafness.
In clinical practice it is common to see patients complaining of hearing loss. It is useful to be able to have a simple test that can distinguish where the site of the cause of the hearing loss is. This can help guide the need for further examination, investigation and management. Rinne’s test and Weber’s test are done to differentiate between a conductive (middle and outer ear causes) and a sensorineural deafness (caused by damage to the cochlea or to the
8th nerve – or its central connections). These tests are always done together. The Rinne test is done first.
The function of the external ear is to collect sounds vibrations form the air and focus these onto the tympanic membrane. These vibrations are then transmitted through the middle ear cavity by the ossicular chain (Malleus, Incus and Stapes). The stapes transmits these vibrations to the cochlea through the oval window (fenestra ovalis).
Sound can also be transmitted through the bones of the skull to the cochlea. The hair cells in the cochlea convert the physical vibrations into action potentials that are transmitted via the nerves in the vestibulo-cochlear (auditory) nerve to the brainstem for further processing.
The Rinne and Weber tests are used for quick screening of conductive versus sensorineural hearing loss. The Rinne test is performed for evaluation of hearing loss in one ear (unilateral hearing loss) and compares perception of sound transmitted by air conduction to those transmitted by bone conduction through the mastoid. Therefore, the presence of conductive hearing loss is screened for.
For the Rinne test, a vibrating tuning fork (typically 512 Hz) is placed initially on the mastoid process behind each ear until sound is no longer heard, signaled by the patient. The fork is then immediately positioned just outside the ear with the patient asked to report when the sound caused by the vibration is no longer heard. A normal or positive Rinne test is when the sound heard outside the ear (air conduction or AC) is louder than the initial sound heard when
the tuning fork end is placed on mastoid process behind the ear (bone conduction or BC). Therefore, AC > BC; which is how it is reported clinically for a normal or positive Rinne result. In conductive hearing loss, bone conduction is better than air or BC > AC, a negative Rinne.
In the Weber test, a vibrating tuning fork (often 256Hz or 512Hz) is placed in the middle of the forehead, or above the upper lip under the nose over the teeth, or on top of the head in an equal distant from the patient’s ears. The patient is then asked to report in which ear the sound is louder. A Weber test result is normal (positive) if the patient reports the sound equally on both sides. Lateralization (hearing the sound better in one ear) is due to either a sensorineural or conductive problem. In a patient with sensorineural hearing loss, the normal ear hears the sound better than the defective one. In a patient with conductive hearing loss, the sound is lateralized to the affected side.
NOTE – Patients with symmetrical hearing loss have also a positive (normal) Weber test result because both ears are affected equally and no lateralization occurs despite the fact that there is hearing loss.
The results of these two tests are compare to localize and characterize the nature of any detected hearing loss.These test are, however, screening tests and are not replacements for formal audiometry.
In this boy, a positive (normal) Rinne test for both ears excludes conductive impairment. The Weber test is also positive for both ears indicating that there is no lateralization. Based on these findings, chronic perforation of the tympanic membrane (option D), chronic secretory otitis media (option E), and otosclerosis (option B) are excluded
from the differential diagnoses. This boy should have a hearing problem affecting both ears equally.
NOTE - Otosclerosis is a disease characterized by fusion of the stapes to incus resulting in conductive hearing loss. This disease is more common among young adult. This boy has normal Rinne test and this condition is unlikely to be the cause of his hearing loss.
Presbycusis (option A) is the most common cause of sensorineural hearing loss in aging individual. A 10-year-old boy is unlikely to have presbycusis.
Given the exam findings and the age of this child, congenital sensorineural hearing loss either in isolation or associated with a syndromic condition could be the most likely diagnosis among others.
A 15-year-old boy is brought by his parents for his behavioral problems. According to the parents, he has had marked aggressiveness with several fights and arguments with the school staff, anger outbursts, and being uneasy to control. He seemingly is not obedient at home and defies his parents and the home rules. Parents say that they have fights over every simple issue at home. They, however, deny any act of vandalism, cruelty towards animals or people,
or drug use. He has declined school performance. On examination, he has a normal attention span and does not appear hyperactive. If pharmacological intervention is required, which one of the following is most likely to benefit him?
A. Carbamazepine.
B. Sodium valproate.
C. SSRIs.
D. Risperidone.
E. Olanzapine.
Correct Answer Is D. Risperidone.
The clinical picture and the age of the child are mostly consistent with oppositional defiant disorder (ODD) as the diagnosis.
ODD is a disruptive behavior disorder in children and teenagers characterized by patterns of unruly and
argumentative behavior and attitudes toward authority figures. This pattern is often considered by parents simply as stubbornness, emotionalism, and strong will. However, in ODD the behavior is significantly more extreme than what is considered normal. Interestingly, ODD occurs much more frequently than the type of childhood stubbornness,
whininess, and rebellion that often occurs at different stages of a child’s development.
ODD manifests with a constant extremely negative, defiant, and hostile behavior leading to disruption of the social, school, and home life for at least 6 months. Symptoms may appear as early as the late preschool years.
Children with ODD often direct their anger and resentment toward their parents, teachers, and other authority figures; however, they can have such problems with their peers as well. They are often uncooperative, vindictive, and easily annoyed. They usually defy the rules, have anger outburst, blame others for their mistakes, seek revenge, and disturb
others on purpose. ODD symptoms may be directed at one person or many people, and may occur only at home, at school, or may occur in a number of settings.
Since the child is unlikely to understand that he has a problem, seeking treatment is often from the parents’ side. A careful history is essential because many other childhood conditions may have some shared features with ODD.
Also, ODD may co-exist with other psychiatric conditions such as anxiety disorder, ADHD, learning disorders and language disorders. It is important to differentiate whether the abnormality in the behavioral pattern is due to ODD or simply a response to a temporary situation.
The international Classification of Diseases 10
th Revision (ICD-10) classifies** ODD as a mild form of conduct disorder**. It has been estimated that up to 60% of patients with ODD will develop conduct disorder.
Treatment is necessary at earlier stages to prevent it from developing into a more serious conduct disorder (most important), mental health disorder, or criminal behavior. Treatment entails a combination of behavioral therapy, family therapy, and at occasions, medications.
In more than 50% of patients with ADHD, ODD is also part of the clinical picture. There is strong evidence suggesting that ODD and ADHD overlap and many medications that are used to treat ADHA may also be efficacious in the treatment of ODD too. A few studies have reported the positive effects of psychostimulants or atomoxetine in the treatment of ODD associated with ADHD. Patients with ODD and conduct disorder with severe aggression may well respond to risperidone, with or without psychostimulants.
Mood regulators, alpha2 agonists, and antidepressants may also have a second-line role in the treatment of ODD and its comorbidities.
Of the options, risperidone is the only option that can be used if pharmacotherapy is considered.
Mood stabilizers such as sodium valproate (option B) or olanzapine (option A), and antidepressants such as SSRIs (option C) are second-line options.
There is no evidence supporting carbamazepine (option E) as a pharmacological treatment for ODD.
A 7-year-old boy presents with a generalized rash illustrated in the following photograph. The rash developed after 24 hours of mild malaise and fever. When he can go back to school?
A. After the resolution of the blisters.
B. After 2 days.
C. After 5 days.
D. No exclusion is required.
E. After clearance of the rash.
Correct Answer: A. After the resolution of blisters.
The photograph shows blisters of varying stages, macules and papules. Of the blisters, some are intact, some unroofed and some dried. This, along with the history, is highly diagnostic for chicken pox.
Children with chickenpox should be excluded from school or other daycare setting until all blisters are dried out. This usually takes 5 days from the onset of the rash but may be less in previously immunized children.
Dried blisters take more time to clear, during which the child is not infectious. Hence, waiting until the complete clearance of the rash is not necessary.
The parents of a 7-year-old girl have brought her to you for assessment because she frequently soils her underwear. In the past 2 months, they have received calls from her school about her problems there. She is easily irritated and becomes angry and aggressive with her classmates and other children at school. At home, they found her soiled underwear hidden. She becomes angry, cries, and fights with her older brother when he calls her ‘smelly’. This happens several times a week. Which one of the following is the most likely diagnosis?
A. Conduct disorder.
B. Oppositional defiant disorder.
C. Regression.
D. Delayed developmental milestones.
E. Depression.
Correct Answer Is B, Oppositional Defiant Disorder.
The scenario suggests encopresis. However, full diagnostic criteria for such diagnosis is uncertain (frequency and duration of symptoms are lacking in the scenario).
Encopresis is the voluntary or involuntary passage of formed, semi-formed, or liquid stool into a place other than the toilet for more than one time per month in a child older than 4 years of age for at least 3 months. If the child has never been continent, the condition is termed primary encopresis, whereas secondary encopresis is fecal incontinence in a previously continent child. Encopresis is reported in 1-4% of school-aged children. Encopresis is
more common in boys than girls.
There are two different types of encopresis:
With constipation and overflow Incontinence: children with the constipation and overflow incontinence type have less than 3 bowel movements per week. Due to constipation, only part of the total available stool is emptied during each of these movements. Parts of the remaining stool leak out, often during the child’s daily activities. Once the constipation is resolved, the encopresis usually no longer exists.
Without constipation and overflow Incontinence: in this type, there is no constipation, and the child’s feces have normal consistency. Unlike in cases associated with constipation and overflow, soiling of this type is intermittent.
Feces may be emitted in a prominent location (e.g., as an act of defiance) or maybe an unintentional consequence of anal self-stimulation (e.g., a variety of masturbation). Encopresis without constipation and overflow incontinence is less common than the first type of encopresis and is often associated with oppositional defiant disorder and conduct disorder. It has been estimated that 3% of children with psychiatric issues may have encopresis.
With aggressiveness, anger outbursts, and being easily annoyed and irritated, oppositional defiant disorder is the most likely underlying cause of encopresis and the diagnosis. This girl has no history of violence, a serious breach of law, or vandalism to suggest conduct disorder (option A).
(Option C ) Regression is a defense mechanism characterized by the reversion of an individual’s personality to an earlier stage of development and adopting more childish mannerisms. In children, regression presents with returning to behaviors that they have already grown out of. Examples are wanting a bottle or pacifier, temper tantrums, whining, aggression, thumb-sucking, and baby talk. Enuresis (bet wetting) or encopresis may occur. Regression is
often triggered by a stressful life event such as bereavement, parental separation, or sexual abuse. In the absence of such history, regression is less likely as the diagnosis. Furthermore, the behavioral pattern of this child is more consistent with oppositional defiant disorder.
(Option D) Encopresis is unlikely to have been caused by delayed development in a girl who has otherwise fulfilled other developmental milestones.
(Option E) There is no strong link between depression and encopresis. Moreover, there are no other symptoms such as low mood, sadness, or altered sleep pattern and appetite to suggest depression as a diagnosis.
A 24-month female child is brought for assessment of development. Which one of the following is expected to develop at this age?
A. Knowing two pronouns.
B. Naming four colors.
C. Knowing her age.
D. Knowing her family name.
E. Speaking in full sentences.
Correct Answer Is A.
Of the options knowing pronouns and prepositions is a language developmental milestone that could have been achieved at this age. Other options are often achieved after the age of** 3** years (36 months)
(Option B) Naming some colors is the ability achieved between** 3 and 5 years** of age. It is too soon for this child for this.
(Option C) knowing the age is an ability achieved often after 5 years of age.
(Option D) Knowing the family name is an expected ability around the age of 4 years. Assessment should be considered if a child cannot say their family name by the age of 5 years.
(Option E) Speaking in full sentences using many words is a language development milestone normally achieved between the ages **3 and 5 **years. It would not be expected in a 24-month-old child.
Parents of a 14-month-old boy has brought him for evaluation because they think their baby lags behind language development milestones because he does not speak any words and only babbles. He is the outcome of an uneventful pregnancy and was born through an uncomplicated vaginal delivery. He has not had any medical problems to date except few episodes of upper respiratory tract infections. He started rolling over at the age of 4 months, can sit without support from the age of 8 months and has started unaided walking recently. He transfers objects from one hand to the other and plays and enjoys peak-a-boo. Which one of the following this child have?
A. Language delay.
B. Social delay.
C. Normal development.
D. Fine motor delay.
E. Gross motor delay.
Correct Answer Is C.
Developmental milestones are categorized under the following developmental areas:
* Physical
* Social
* Emotional
* Cognitive
* Language
This child has been able to roll over since the age of 4 months (normal: 0-4 months), can sit without support from the age of 8 months (normal: 8-12 months), can transfer objects from one hand to the other (normal: 8-12 months) and has started walking without support (normal: 1-2 years). In terms of physical development, he is fully developed and does not have any fine motor (option D) or gross motor (option E) developmental delay. He plays peak-a-boo (normal: 4-8 months). This is an example of cognitive developmental milestone often achieved between 4-8 months of age.
There is no information regarding social developmental milestones in the question but parents do not seem to be concerned about it. This exclude social delay (option B) as his problem as parents are first to suspect something wrong in their baby and they are often right for that matter.
In fact the parent’s concern is only language delay.
A child aged 1 to 2 years is expected to:
* Say first name
* Say many words (mostly naming words)
* Begin to use one- to two-word sentences such as ‘want milk’
This child is 14 months and have enough time to fulfill the above milestones. He has been developing normal so far and has not lagged behind. Language delay would be a concern if the child still babbles beyond 24 months of age.
You order an ultrasound scan for a 9-year-old girl, who has been brought for treatment of urinary tract infection for the second time this year. On the ultrasound, the right kidney is reported smaller than the left beyond the normal discrepancy. Which one of the following is the most appropriate investigation to consider for assessment of renal status?
A. DMSA.
B. DTPA scintigraphy.
C. Urea and electrolytes.
D. Abdominal CT scan.
E. Urine culture.
Correct Answer Is A, DMSA.
The dimercaptosuccinic acid scintigraphy (DMSA) scan is the gold standard for diagnosis of kidney scarring, which can have been resulted from recurrent urinary tract infections.
DMSA is indicated in the following conditions:
1. 1. Clinical suspicion of renal injury
1. Reduced renal function
1. Suspicion of VUR
1. Suspicion of obstructive uropathy on ultrasound in older toilet-trained children
For this child with recurrent episodes of UTI and a smaller-than-normal kidney, DMSA should be considered as the most diagnostic modality for assessment of possible renal scarring.
(Option B) DTPA scintigraphy is a type of radioisotope renography using DTPA and **Tc99 **as radio-labeled material.
This test also provides information regarding renal function and/or scarring. Compared with DMSA, this test is faster and associated with less radiation but DMSA remains the gold standard criterion for assessment of renal scarring.
(Option C) Urea and creatinine abnormalities are common among patients with urinary problems. Elevated levels of urea and creatinine indicated renal impairment but gives no clue regarding the kidney function per se. Urea and creatinine may raise in patients with pre-renal renal failure or post-renal renal failure in the initial course of which the
kidneys are normal or near normal. On the other hand, normal values do not exclude the presence of abnormalities in a single kidney. Many patients may have normal ranges in the presence of severe damage to one kidney while the other is functional and compensating.
(Option D) Abdominal CT scan will visualize the kidneys, adrenal glands and adjacent structure. It is not capable of assessment of renal function.
(Option E) Urine culture will show the presence or absence of infection in kidneys but does not provide any clue regarding current kidney function.
A five-year-old boy is brought for evaluation by his parents concerning unusual behavior. He loves Popeye cartoon and always remembers when it is shown on TV. At that time he turns on the TV. He becomes distraught and irritated if he misses the cartoon. He has a goldfish and spends several hours staring at it with curiosity and fascination. His
favorite toy is wooden building blocks that he uses to make towers, break them and build them again. He did not say his first word until the age of 2.5 years and could not use a communicative phrase at the age of 3 years. He does not play with his peers and is not interested in making friends. During the examination, he adamantly avoids eye contact.
Which one of the following is the most likely diagnosis?
A. Asperger syndrome.
B. Autism.
C. Mental retardation.
D. Delayed developmental milestones.
E. Obsessive-compulsive disorder.
Correct Answer Is B, Autism.
The persistent ritualism and preoccupations with activities (watching the Popeye cartoon) and objects (the building blocks and the goldfish), delayed speech language development, poor communication with friends and avoiding eye contact make Autistic disorder the most likely diagnosis in this child.
Autistic disorder is pervasive developmental disorder (PDD) affecting at least 4 children in 10000 with a male to female ration of 4:1.
Characteristic features of autistic disorder include the following:
1- Onset during infancy and early childhood
2- An impairment of social interactions shown by at least two of the following:
* lack of awareness of the feelings of others
* absent or abnormal comfort seeking in response to distress
* lack of imitation
* absent or abnormal social play
* impaired ability to socialize, including avoiding eye contact
3- Impairment in communication as shown by at least one of the following:
lack of babbling, gesture, mime or spoken language
absent or abnormal non-verbal communication
abnormalities in the form or content of speech
poor ability to initiate or sustain conversation
abnormal speech production
4- **Restricted or repetitive **activities, interests and imaginative development, shown in at least one of the following:
* stereotyped body movements
* persistent and unusual preoccupations and rituals with objects or activities
* severe distress over changes in routine or environment
* an absence of imaginative and symbolic play
5- Behavioral problems:
* tantrums
* hyperactivity
* destructiveness
* risk-taking activity
In a nutshell diagnosis of autistic disorder requires the presence of the 3 following core features by the age of 3 years:
1. Qualitative impairment of** social interaction**
2. Qualitative impairment of communication
3. Restricted, repetitive and stereotyped patterns of activities, behavior and interest
A majority of children with autistic have intellectual disability. Approximately 30% of autistic children have normal intellectual development.
(Option A) Asperger syndrome, also called high-functioning autism, shares many features with autistic disorder. However, children with Asperger syndrome have normal language development and intellectual ability. With language problems in this child, Asperger syndrome is an unlikely diagnosis.
(Option C) Although intellectual disability is common in children with Autism, this is a condition not a diagnosis.
(Option D) Children with Autism have language and cognitive problems but again these are specific conditions not a diagnosis. On the other hand, language development is impaired not delayed. ‘Delayed’ implies that such milestones may be reached at a later stage which will not happen in autistic children.
(Option E) Routines and ritualistic behavioral patterns are features seen in obsessive-compulsive disorder (OCD); however, delayed language, stereotypic behavior, and poor communications skills present in this child make OCD an unlikely diagnosis.
A 4-year-old girl is brought to you by her kindergarten teacher. She states that the child has not been well since this morning, vomited once, and complained of central abdominal pain. On examination, the child looks unwell and has a temperature of 39°C. Which one of the following would be most expected in this child?
A. Numerous pus cells on urine microscopy.
B. Inflamed ear drums.
C. Neck stiffness.
D. Inflamed tonsils.
E. Localized tenderness over the right iliac fossa.
Correct Answer Is A.
It is not uncommon for young children to have urinary tract infection (UTI) without classic urinary symptoms of urgency, frequency and/or dysuria. In fact, the younger the child, the more likely the symptoms are vague and nonspecific. In case the cause of such presentation is a UTI, abnormally high counts of white blood cells (puss cells) are the most common and expected finding on urinalysis.
(Option B) Inflamed ear drums are seen in otitis media. Otitis media is not associated with abdominal pain; therefore, not a likely diagnosis with this constellation of symptoms.
(Option C) Neck stiffness could be elicited in meningitis. Fever and unwellness are expected features, but abdominal pain is less likely.
(Option D) Tonsillitis can make a child ill and febrile, but does not cause abdominal pain.
(Option E) Central abdominal pain, vomiting and localized tenderness over the right iliac fossa can be caused by appendicitis, but it is uncommon for appendicitis to cause a fever as high as 39°C, unless it is complicated due to perforation. Since the symptoms has commenced this morning, perforation of an appendicitis would not be expected. It is unusual for an appendicitis to perforate this early.
A 7-year-old boy weighs 30 kg and has a body mass index of 19.5 kg/m2. Which one of the following is correct regarding his weight and BMI (use the following chart for calculation)?
A. He is underweight.
B. He is overweight.
C. He has normal weight.
D. He is obese.
E. BMI is not applicable in children.
Correct Answer Is D.
Since the body composition changes with normal growth and stage of puberty, BMI in children is not a fixed measure like it is in adults. BMI interpretation in children 2-18 years of age must be plotted on the sex-specific BMI percentile charts. A 17-year-old boy with a BMI of 20 kg/m2 is within the normal range while a 6-year-old boy with the same BMI is obese.
To check the weight status of a child, his/her BMI should be calculated by dividing the weight by the square of the height in meter. The BMI then should be plotted on the sex-specific BMI percentile to see in which zone it is. This child’s BMI lies in the ‘obesity’ zone (see the chart below).
If used with the sex-specific percentile chart, BMI can be used for children with the same efficacy as for adults.
Rachel, 5 years old, is brought to your GP clinic by her mother because of pain and redness in the back of her left ear.
According to the mother, Rachel developed fever and left ear ache one week ago, for which he was seen by another GP and started on paracetamol and amoxicillin. She was feeling better for the first 4 days but spiked a fever again and developed a painful red swelling behind her left ear. On examination, she has a warm, red, and tender swelling behind the left ear shown in the accompanying photograph. Which one of the following could be the most likely pathogenic factor for this presentation?
A. Staphylococcus aureus.
B. Streptococcus pneumoniae.
C. Hemophilus influenzae.
D. Moraxella catarrhalis.
E. Streptococcus viridans.
Correct Answer Is B.
With the history of unilateral earache and fever one week ago, and development of a red and tender swelling behind the affected ear, acute mastoiditis complicating acute otitis media (AOM) is the most likely diagnosis.
Acute mastoiditis is rare; however, it is the most common suppurative complication of AOM. Acute mastoiditis may result in intracranial complications; therefore, it should be taken very seriously. Treatment of acute mastoiditis starts with intravenous antibiotics (Flucloxacillin plus a third-generation cephalosporin). Immediate involvement of an ENT consultant is of paramount importance as some cases may even require surgical intervention.
Acute mastoiditis is diagnoses based on postauricular (behind the ear) inflammatory signs such as erythema, edema, tenderness, and fluctuance. Other clinical findings include edema of auricle and/or external canal and associated signs of AOM. Although acute mastoiditis follows AOM, it can be the first presentation of AOM in some cases.
Common pathogens for acute mastoiditis are the same for AOM, with streptococcus pneumoniae being the most common one and the cause in over 50% of cases. Hemophilus influenza (option C) is the second most common causative organism responsible for such presentation.
Staphylococcus aureus (option A), Moraxella catarrhalis (option D), and streptococcus viridans are less common pathogenic organisms in AOM and acute mastoiditis.
A 5-year-old child is brought to your practice by his parents with complaints of fever and ear pain. On examination, he is in mild distress and has a fever of 38.3°C. Otoscopic examination reveals a red bulging tympanic membrane on the right side with decreased mobility. Which one of the following is** more likely to have caused** this presentation?
A. Hemophilus influenza.
B. Streptococcus pneumoniae.
C. Pseudomonas aeruginosa.
D. RSV virus.
E. Moraxella catarrhalis.
Correct Answer Is B.
A red bulging tympanic membrane with decreased mobility is the classic finding in acute otitis media (AOM).
In children older than 6 weeks, AOM is caused by bacteria in majority of cases. Streptococcus pneumoniae, Hemophilus influenzae, Moraxella catarrhalis, and Streptococcus pyogenes are responsible for the majority of episodes of AOM in persons older than 6 weeks. Other bacteria implicated in AOM include Staphylococcus aureus,
streptococcus viridans, and Pseudomonas aeruginosa.
Of these, Streptococcus pneumoniae is the most common etiologic agent responsible for AOM and for invasive bacterial infections in children of all age groups, followed by Hemophilus influenzae as the second most common etiologic factor.
RSV virus is the etiologic factor, mostly in neonates. Infection with this virus is associated with pneumonia,
bronchiolitis and otitis media in this age group; however, streptococcus pneumoniae remains the most common etiologic factor for AOM even in this age group.
Parents of an 11-year-old boy have brought him to your clinic for assessment because they believe he is obese. On examination, his weight and height are on 90 and 50 percentiles for sex- and age-matched growth charts respectively. Which one of the following is the most appropriate investigation to consider for him?
A. GH.
B. FSH and LH.
C. TSH.
D. Bone age.
E. Reassure he has normal growth and investigation is required.
Correct Answer Is C
For children and adolescents aged 2–18 years, growth is monitored based on age, height, and weight, using sexspecific Body Mass Index (BMI) percentile charts. BMI is not a fixed measure in this age group but varies with normal growth, stage of puberty, and sex. Either the United States Centers for Disease Prevention and Control (US-CDC) or WHO BMI percentile charts may be used, with the same chart used over time to allow for consistent monitoring of
growth. The US-CDC categorizes overweight as between the 85th and 95th percentile and obesity as above the 95th percentile. The WHO categorizes overweight as between the 85th and 97th BMI percentiles and obesity as above the 97th percentile. These categories are not diagnostic but contribute to the overall clinical impression of the child or adolescent being measured.
This boy, based on either chart, is overweight (>85th). The likelihood that childhood overweight and obesity will persist into adulthood increases with the age of the child and with the presence of parental obesity. One of the strongest predictors of a child’s weight is the weight status of his/her parents. For overweight or obese children, initial assessment by history and clinical assessment should determine current health problems and risks for future disease.
History taking includes developmental history, physical and mental health (including family history of obesity), and current health behaviors.
Clinical assessment includes pubertal stage, possible causes for overweightness or obesity (e.g., hypothyroidism), and indicators of comorbidities (e.g., raised blood pressure, joint pain, gastrointestinal symptoms, insulin resistance, intertrigo, dental health etc.)
It is also very important that secondary causes for obesity are considered and excluded. Of the options, TSH is an appropriate investigation for exclusion of hypothyroidism as a potential cause of overweightness.
(Options A and B) There is no clinical finding to suggest FSH/LH or GH as an initial assessment. Such clinical findings may include but not limited to short stature (necessitating GH evaluation) and delayed puberty (for TSH, FSH and LH).
(Option D) Bone age is often used as the initial assessment for short stature that is not the problem here.
(Option E) This boy is overweight and reassurance cannot be given because there might be an underlying cause present and also the child is at increased risk of obesity-relates health problems in the future.
Which of the following organism is the most common cause of urinary tract infections in children?
A. Proteus mirabilis.
B. E.coli.
C. Staphycoccis aureus.
D. Enterobacter.
E. Pseudomonas.
B. E.coli
In children, between 75 to 90% of urinary tract infections are caused by E.coli. Other organisms may include Klebsiella pneumonia, proteus mirabilis, staphylococci species, pseudomonas and enterobacters.
- Medscape - Pediatric Urinary Tract Infection
- RCH - Urinary tract infection
A 15-year-old girl presents with an upper respiratory tract infection (URTI). On examination, she has a temperature of 38.1°C and an erythematous pharynx with no exudate. She is otherwise healthy. A urine dipstick is positive for blood for which you order a urinalysis (UA). UA is reported back significant for red blood cells, 1+ proteinuria, no WBCs, and negative for nitrite. Which one of the following would be the next best step in management?
A. Repeat UA after the URTI resolves.
B. Urine culture.
C. 24-hour urine exam.
D. Ultrasonography.
E. Start her on corticosteroids.
A. Repeat UA after the URTI resolves
Asymptomatic hematuria in children, in the setting of a febrile illness other than urinary tract infection, should be assessed after the febrile illness subsides; provided that the illness is not due to urinary tract infection (UTI). With UTI, the next step would be a urine culture. This patient neither has urinary symptoms, nor laboratory findings consistent with UTI; therefore, no urine culture is indicated.
Asymptomatic microscopic hematuria should always be confirmed with 2 positive urine exams out of 3 in 2-3 weeks before any further assessment is considered. However, in the presence of symptoms such as hypertension, edema, or renal failure, prompt action is mandated.
A 5-year-old boy is brought to the Emergency Department by ambulance after he had a generalized tonic-clonic seizure at home 20 minutes ago. Upon arrival to the hospital, he was assessed immediately. He is not having a seizure now but is lethargic and confused. His blood pressure is 105/65 mmHg, pulse rate 76 bpm, respiratory rate 16 breaths/min, and temperature 37°C. His mucous membranes are not dry, skin turgor is normal, and capillary refill time is 2 seconds. The rest of the examination, including cardiovascular, respiratory and neurological exam is completely normal. Pathology results are as following:
* FBE: Normal
* Random blood sugar: 8.3 mmol/L (4 - 11.1 mmol/L)
* Sodium: 120 mmol/L (135-145 mmol/L)
* Potassium: 4.1 mmol/L (3.5 – 5.5 mmol/L)
* Bicarbonate: 24 mmol/L (22-32 mmol/L)
* Creatinine: 80 μmol/L (60-110 μmol/L)
* Urea: 4.5 mmol/L (2.5-7.1 mmol/L)
* Calcium: 3.1 mmol/L (2.2-2.7 mmol/L)
Which one of the following could be the most likely cause to this presentation?
A. Acute renal failure.
B. Addison disease.
C. Congestive heart failure.
D. Hyponatremia due to dehydration.
E. SIADH syndrome.
E. SIADH
Seizure can be caused by a variety of causes including:
* Metabolic derangements such as hyponatremia, hypernatremia, hypocalcemia, and hypoglycemia
* Structural abnormalities such as space occupying lesions in the brain
* CNS Infections such as meningitis
* Medication intoxication or withdrawal
* Epilepsy
There are a few clues in the history, physical examination, and laboratory findings that point towards hyponatremia as the most likely explanation for this presentation. Hyponatremia (and hypernatremia) present with CNS manifestations including lethargy, headache, alteration in consciousness, and in worse cases, seizure, coma, or even death. He also has hypercalcemia. But it is it is uncommon for hypercalcemia to cause seizures while hypocalcemia is notorious for that.
All the given options can potentiate and cause hyponatremia; hence, such presentation. However, this child has no signs of dehydration, and this excludes hyponatremia due to dehydration (option D) as a cause. Also, there is no history of congestive heart failure option C) nor any clinical findings such as abnormal cardiovascular exam findings, volume overload, etc. to support such diagnosis.
Acute renal failure (option A) can also cause hyponatremia but the normal urea and creatinine rules out such diagnosis.
Addison disease (option B) can be another explanation for hyponatremia in general but not as a likely diagnosis in this scenario. In Addison disease, adrenal insufficiency and lack of aldosterone result in hyponatremia and hyperkalemia at the same time. This child has a normal potassium. Furthermore, hypotension is one of the main clinical findings in patients with Addison disease. This child has normal blood pressure. Collectively, these make Addison disease a very remote possibility.
Of the options, SIADH is most likely to have resulted in such presentation. SIADH (syndrome of inappropriate anti- diuretic hormone) is caused by excess amount of anti-diuretic hormone (ADH) from the hypothalamus-pituitary unit. ADH affects kidneys to concentrates urine and retain water. In SIADH, excess ADH results in a very concentrated urine, increased body water, and hyponatremia through dilutional effect.
Depending on the rate at which hyponatremia occurs, patients with SIADH can be asymptomatic or have classic symptoms of hyponatremia including the following:
* Nausea and vomiting
* Headache
* Problems with balance that may result in falls
* Mental changes, such as confusion, memory problems, strange behavior Seizures or coma, in severe cases
Common causes of SIADH are:
* Medicines, such as certain type 2 diabetes medications, anti-epileptic medications, antidepressants, heart and blood pressure drugs, cancer drugs, drugs used for anesthesia
* Surgery under general anesthesia
* Disorders of the brain, such as injury, infections, stroke
* Brain surgery in the region of the hypothalamus
* Lung disease, such as pneumonia, tuberculosis, cancer, chronic infections
* Cancer of the lung, small intestine, pancreas, brain, leukemia
- RACGP – AFP – The suspect: SIADH
Concerned parents of a 6-year-old girl has brought her to your GP clinic as they have noticed scattered growth of pubic hair in their daughter recently. On examination, she has sparse pubic hair growth and Tanner I breast. Her height and weight are on 75% and 50% percentiles respectively. Which one of the following is the most appropriate next step in management?
A. Ultrasounds scan of the pelvis.
B. Assessment of the bone age.
C. FSH and LH.
D. Review in 3 months.
E. 17-hydroxyprogesterone and estradiol.
D. Review in 3 months
The scenario represents development of pubic hair before the age of 8 years in girls (or 9 in boys) in the absence other sexual characteristics which are thelarche (breast development) and menarche (periods), which is referred to as premature isolated adrenarche. Adrenarche is the maturation of the adrenal zona reticularis in both boys and girls, resulting in the development of pubic hair, axillary hair, and adult apocrine body odor. Premature isolated adrenarche is considered a benign variant of normal development if the weight and height are normal for the age.
No investigations are required for premature isolated adrenarche as FSH and LH (option C), 17-hydroxyprogesterone (option E) and estradiol , and testosterone all normal. The only laboratory finding might be slight elevation of dehydroepiandrosterone sulfate (DHEAS). It is recommended that children with premature isolated adrenarche are followed every 3-6 months for surveillance of puberty development and linear progression and velocity of growth.
Bone age assessment (option B) is considered in constitutional growth delay (CGD) alongside exclusion of other causes of delayed puberty/growth to confirm the diagnosis or where the child is small for age. This child has normal weight and height for age.
Ultrasound of the pelvis (option A) is the study of choice for girls whose periods are overdue despite development of other secondary sexual characteristics, normal body measures, and absence of features suggesting chromosomal abnormalities such as Turner syndrome.
NOTE - puberty starts with thelarche. A Tanner II breast before the age of 13 years suggests estrogen exposure and commencement of puberty in girls. This is equivalent to testicular enlargement in boys. A Tanner II or less after 13 years is abnormal and indicated absence of breast development.
- American Family Physicians: Disorders of Puberty: An Approach to Diagnosis and Management
- RACGP – AFP – Growth disorders in children
Concerned parents of a 5-year-old boy have brough him to the Emergency department because he has been having palpitations and dizziness since this morning. He is otherwise healthy with no significant medical history, and this is the first time he is experiencing it. An ECG as well as blood tests are arranged. On examination, he has a weak thready pulse which is so rapid that cannot be counted correctly. His blood pressure is 98/60 mmHg. He seems restless but does not seem to be breathless. Extremities are of normal temperature and color. A 12-lead ECG is obtained and is shown in the following photograph. Which one of the following is the most appropriate next step in management of him?
A. Immersion of face in cold water.
B. Intravenous adenosine.
C. Intravenous amiodarone.
D. Synchronized cardioversion.
E. Intravenous esmolol.
C. Intravenous amiodarone
The ECG shows wide-QRS complex tachycardia (WCT) at a rate of approximately 300 bmp. Most WCTs in children are in fact supraventricular tachycardias (SVT) with aberrancy. Compared to SVT with aberrancy, ventricular tachycardia (VT) is rare in children; however, due to seriousness of the condition, Australian Advanced Paediatric Life Support (APLS) and Royal Children Hospital (RCH) guidelines recommend that WCTs are considered and treated as VT until proven otherwise. This approach is different from recommendations by American Heart Association (AHA) that advise to treat WCTs in a hemodynamically stable children as SVT first by a trial of intravenous adenosine (option B) before making a diagnosis of VT.
APLS recommends intravenous amiodarone under close monitoring for treatment of symptomatic VT in children with stable hemodynamics, and synchronized DC cardioversion in those with hemodynamic instability/ shock if there is pulse and defibrillation in the absence of a pulse.
This child is hemodynamically stable; therefore, intravenous amiodarone will be the next best step in management.
OPTION A : Vagal maneuvers such as immersion of the face in cold water , application of ice pack to the face or Valsalva maneuvers are initial treatment for patients with narrow-QRS complex tachycardia i.e., SVT. Vagal maneuvers suppress the AV node and might worsen the condition in VT.
OPTION E : Intravenous beta blockers such as esmolol are not generally recommended for treatment of VT or SVT in children.
- Advanced Paediatric Life support (APLS) – Algorithms – Ventricular Tachycardias
- Medscape – Pediatric Ventricular Tachycardia Overview of Ventricular Arrhythmias
Emad is 6 years old and is being rushed to the Emergency Department by an ambulance after he experienced light- headedness, shortness of breath, chest tightness, and anxiousness at home 15 minutes ago. When you visit him, he is lying on a stretched and is receiving oxygen by non-re-breathable face mask. He is mumbling and does not seem to be oriented to time and place however he seems to recognize his parents. Examination reveals a very rapid barely perceptible pulse and blood pressure of 80/45 mmHg. Cardiac monitoring shows the following rhythm. Which one of the following is the most appropriate immediate management in this situation?
A. Defibrillation.
B. Synchronized DC cardioversion.
C. Intravenous amiodarone.
D. Intravenous lignocaine.
E. Intravenous adenosine.
B. Synchronized DC cardioversion
Emad’s ECG shows tachycardia of approximately 300 bpm with regular wide QRS complexes (wide-QRS complex tachycardia [WCT]). In children, ventricular tachycardia (VT) is very rare, and most WTCs are in fact supraventricular tachycardia with conduction aberrancy (SVT with aberrancy). However, due to seriousness of VT compared to SVT, guidelines such Advanced Paediatric Life Support (APLS) as those by the Royal Children Hospital (RCH) recommend that WCTs should be considered and treated as VT until prove otherwise.
As such, Emad’s ECG should be considered and treated as VT. Treatment of VT with a pulse is determined by the presence or absence of shock/hemodynamic instability (hypotension, hypoperfusion, altered mental status)
For every patient presenting with hemodynamic instability caused by a cardiac tachyarrhythmia, DC cardioversion should always be considered as the most appropriate treatment option (defibrillation for ventricular fibrillation and pulseless VT; synchronized for the rest.)
Emad has an ECG with WCT in addition to hypotension and hypoperfusion evident by altered mental status (disorientation to time and place) and cold pale sweaty extremities. Of the options, synchronized DC cardioversion is the next step in treatment to consider for him.
Defibrillation (unsynchronized) (option A) is used as first line for treatment of ventricular fibrillation (VF) and pulseless VT.
If Emad was not hemodynamically unstable, intravenous amiodarone (option C) would be considered first. Amiodarone is the medication of choice for children presenting with symptomatic wide QRS tachycardia who are not in shock.
Intravenous amiodarone (class III antiarrhythmic) is almost always the medication of choice for treatment of symptomatic VT in stable infants and young children. The medication has a broad efficacy and is readily available. However, major adverse reactions have always been concerning.
Lignocaine (lidocaine) (option D) is a class I (sodium channel blocker) antiarrhythmic medication. It is no longer recommended for treatment of VT in children.
NOTE - American Heart Association recommends a trial of intravenous adenosine in symptomatic yet stable children with WCT because most of such WTCs are in fact SVT with aberrancy. This way, the child is not unnecessarily exposed to adverse effects of amiodarone or other antiarrhythmics. The Australian Advanced Paediatric Life Support (APLS) and Royal Children Hospital (RCH) guidelines on the other hand endorse considering and treating WCTs as VT rather than SVT until proven otherwise. In any case, **Emad is unstable, and adenosine has no role in his management **based on either of those sets of guidelines.
- APSL – Ventricular tachycardia algorithm
Parents of a 4-year-old girl has brough her to the Emergency Department after she started feeling short of breath and dizzy half an hour ago at home. Parents deny any other previous similar episodes or any significant recent or past medical conditions. On examination, a very rapid thready and hardly perceptible pulse is felt. She has a blood pressure of 98/65 mmHg and respiratory rate of 35 breaths per minute. He is afebrile and has normal peripheral perfusion. You start oxygen via non-rebreathable face mask, put him on cardiac monitoring, and stablish an intravenous line. Cardiac monitoring shows the following rhythm. Which one of the following is the most appropriate next step in management?
A. Put an icepack on his face.
B. Immerse his head in cold water.
C. Intravenous adenosine.
D. Intravenous verapamil.
E. Synchronized DC cardioversion.
A. Put an icepack on his face
The photograph is typical for supraventricular tachycardia (SVT) at a rate of approximately 170 bpm. SVT is an umbrella term used to describe tachycardias (atrial and/or ventricular rates above 100 bpm at rest) that originates from the His bundle or above. These SVTs include inappropriate sinus tachycardia, atrial tachycardia (AT) including focal and multifocal AT, macro-reentrant AT (including typical atrial flutter), junctional tachycardia, atrioventricular reentrant tachycardia (AVRT), atrioventricular nodal reentrant tachycardia (AVNRT), and various forms of accessory pathway-mediated reentrant tachycardias e.g., Wolff-Parkinson, White syndrome (WPW).
In younger children, SVT is often caused by AVRT, including WPW while in adolescents it is caused by AVNRT. Other causes are very rapid sinus tachycardia, atrial flutter, ectopic atrial tachycardia, and junctional ectopic tachycardia.
SVT typically has a fixed rate, usually >220 bpm. Onset and offset are abrupt, and p-waves are either not visible or seen after the QRS complexes. Conditions such as sepsis, pain, dehydration, anxiety, and fever can contribute to development of SVT and should always be considered and addressed in management of children with a tachyarrhythmia
NOTE - SVT typically causes narrow complex tachycardia; however, in SVT with aberrancy, QRS complexes could be wide, resembling ventricular tachycardia (VT). Although the majority (95%) of wide-QRS complex tachycardias in children are SVT with aberrancy, sustained wide-QRS complex tachycardia should always be considered and treated as VT until proven otherwise (RCH recommendation).
Symptomatic SVT often presents with palpitations, shortness of breath, dizziness, and decreased exercise tolerance. If protracted, signs and symptoms of congestive heart failure can develop.
Management of children (and adults) with SVT depends on the presentation. For all patients, oxygen, securing intravenous access, cardiac monitoring, and an a 12-lead ECG should are the initial steps to take. The rest of the management is determined by the presence or absence of indicators of hemodynamic instability which are hypotension, hypoperfusion, or hypoperfusion-related altered mental status.
In the absence of hemodynamic instability, vagal stimulation maneuvers are tried first:
- Neonates and infants (<6 months):
Immersion of the face in ice water for 5 seconds to elicit diving reflex (not to be use for those with hemodynamic instability/shock). While attached to cardiac monitor and arms are wrapped in a towel, immerse the whole face in ice water for five seconds. It is unnecessary to occlude the nostrils. This technique is safe and 90% effective in terminating SVT. - Older infants/ toddlers:
Apply an ice-cold facecloth or bag filled with ice to the face for 15-30 seconds (elicits the ‘dive reflex’). It is less effective than facial immersion but better tolerated in older infants. - School-aged children:
Valsalva technique : with the child in the supine position, ask them to blow on their thumb or a 10 mL syringe, after full inspiration, for 10-15 seconds. There should be no air escape and the child should be seen to strain. In modified Valsalva, the same manoeuvre is performed in a semi-recumbent position. Then at the end of the strain, the child is immediately repositioned into a supine position with passive leg raise to 45 degrees.
This child has symptomatic SVT but is hemodynamically stable. For her, vagal maneuvers are likely to end the SVT and should be tried first. At this age group, putting an ice pack or ice-cold towel/cloth on the face for 15-30 seconds is the preferred method.
Immersion of face (option B) in ice water for five seconds is more effective than placing ice pack on the face. However, it is less likely to be tolerated by children older than 6 months and is not recommended for this age group as the preferred method despite higher efficacy.
Intravenous adenosine (option C) is the preferred medication for terminating SVT in children and adults, but it should be considered if vagal maneuvers fail to work.
Despite the increasing use of the calcium-channel blocker verapamil (option D) for treatment of SVT in adults, it is not routinely used for such purpose in children. It is also contraindicated in children less than 12 months because it can cause irreversible hypotension and fatal ventricular dysfunction in this age group.
Synchronized DC cardioversion (option E) would be the correct answer if the child was in hemodynamic instability/shock. However, some guidelines including Advances Paediatric Life Support (APLS) recommend vagal maneuvers are tried while preparation for DC cardioversion is made, provided it does not cause any delay in delivering the latter.
NOTE - American Heart Association (AHA) recommends that based on rarity of ventricular tachycardias (VT) in children, wide-complex QRS tachycardias with a pulse be considered and treated as supraventricular tachycardia (SVT) with aberrancy if the QRS complex are regular and a pulse is present. Based on recommendations by the AHA (and also UpToDate) a trial of adenosine in such cases should be considered first for children who are hemodynamically stable, both as therapeutic and diagnostic measures (cessation of tachycardia in response to adenosine favors SVT with aberrancy rather than VT). With no response to adenosine, the tachycardia should be treated as VT (e.g., with IV amiodarone).
- RCH – Supraventricular tachycardia (SVT)
- APLS – Supraventricular tachycardia algorithm
Sarah has brought Jesse, her 4-year-old son, Jesse, because of bedwetting. Jesse is her first child and is the outcome of an uncomplicated pregnancy with good health so far. He mingles with his peers and has met all milestones otherwise. He is very attached to his Teddy bear and will not sleep without it. He also has some imaginary friends to whom he talks sometimes. He is dry during the daytime but wets the bed a few times per month. Physical examination is normal. Which one of the following is the most likely diagnosis?
A. Autism.
B. ADHD.
C. Psychosis.
D. Normal development.
E. Anxiety.
D. Normal development
Bedwetting happens when the bladder empties involuntarily during sleep, most children will gain daytime urinary continence by the age of 3 years, and night continence by the age of 6 years. Usually, no treatment is required for night-time bed wetting by the age of 6 years as there is a high rate of spontaneous resolution. Also, attachment to toys or objects such as the Teddy bear in this scenario is completely normal up to the age of 7 years and is a healthy form of play for children.
Jesse is 4 years old and otherwise normal. Sarah should be advised that for now, it can be normal to wet the bed a few nights in a month and that his talking to imaginary friends and attachment to his toys are not concerning at this age and there is no need to be alarmed.
OPTION A : Autism presents with communication problems such as finding friends and playing with peers and repetitive ritualistic behaviors, none of which are present in Jesse.
OPTION B : ADHD presents with poor concentration and impulse control. The main clinical presentation of ADHD includes:
- Inattention: difficulty concentrating, forgetting instructions, moving from one task to the other without completing the previous one
- Impulsivity: acting without thinking, talking over the top of others, losing control of emotions easily, and being accident prone
- Overactivity: constant fidgeting and restlessness
None of the above is present in Jesse.
OPTION C : Having imaginary friends and talking to or playing with them is quite normal in children and part of their social development; therefore, a diagnosis of psychosis cannot be based on that unless other features are psychosis are present e.g., hallucinations, delusions, thought process problems, apathy, etc.
OPTION E : Anxiety presents with affective symptoms such as subjective discomfort, erratic concentration, or hypervigilance or somatic manifestations such as tachycardia, hyperventilation, nausea, vomiting, urinary frequency, or so on. Jesse doe does not seem to have any signs or symptoms suggestive of anxiety.
- RCH – Enuresis: Bed wetting and Monosymptomatic Enuresis
A 35-year-old woman gives birth to her second male baby through an uneventful vaginal delivery. The baby is preterm
at 34 weeks and weighs 3100 gr. Apgar’s scores at the first and 5th minutes are 6 and 9, respectively. Almost one hour after birth, he develops tachypnea and grunting as well as intercostal recession. He is placed under the oxygen hood. In the next 4 hours, his condition remarkably improves. Which one of the following could be the most likely diagnosis?
A. Meconium aspiration.
B. Birth asphyxia.
C. Transient tachypnea of the newborn.
D. Respiratory distress syndrome.
E. Tension pneumothorax.
C. Transient tachypnea of the newborn
The scenario represents early-onset respiratory distress in a newborn with a near normal and normal APGAR score of 6 and 9 at 1 and 5 minutes. These scores exclude birth asphyxia , which is defined as failure to establish breathing at birth. This baby started breathing right after birth but developed respiratory problems shortly after.
Of the other options, transient tachypnea of the newborn (TTN) and respiratory distress syndrome (RDS) seem more likely compared to others.
Transient tachypnea of the newborn (TTN) is a benign, self-limiting condition that can present in infants of any gestational age shortly after birth. The pathophysiology is a delay in the clearance of fetal lung fluid after birth, which leads to ineffective gas exchange, respiratory distress, and tachypnea. RDS on the other hand is caused by surfactant deficiency in preterm newborns.
Both TTN and RDS present with the following:
1. Onset within the first few minutes to hours after birth.
2. Physical exam findings usually include signs of respiratory distress:
-Tachypnea (respiratory rate greater than 60 per minute) Nasal flaring
-Grunting
-Intercostal/subcostal/suprasternal retractions
The distinctive feature between these two is the clinical course. While TTN improves with supportive treatment usually within 6 hours of onset, RDS worsens over 48-72 hours before it improves. The fact that this newborn has made a remarkable recovery in 4 hours makes TTN more likely of a diagnosis compared to RDS.
Meconium aspiration syndrome is a common cause of early-onset neonatal respiratory distress in near- term, term or post-term neonates. One important clue to such diagnosis is the presence of meconium on the neonate’s body (meconium-stained baby) or in the amniotic fluid. Prematurity and absence of any clues on the presence of meconium in the scenario are less likely.
OPTION D : Tension pneumothorax presents with acute onset respiratory distress. Diagnostic clues include tachycardia, tachypnea, hypotension, asymmetrical respiratory movements, absence of breath sounds, and hyper- resonance of the affected side and engorged neck and forehead veins. As none is mentioned in the scenario, it is less likely of a diagnosis.
- Star Pearls – Transient Tachypnea of the Newborn
- Safe Care Victoria – Respiratory Distress Syndrome (RDS) in Neonates
- Safe Care Victoria – Meconium Aspiration Syndrome
- RCH – Recognition of the seriously unwell neonate and young infant
- AAFP - Newborn Respiratory Distress
A 9-year-old girl was brought to your practice two weeks ago with a fever. Your assessment established an upper respiratory tract infection (URTI) as the diagnosis; however, you were also concerned about 3+ blood and 1+ proteinuria on a dipstick urine exam; therefore, you advised a follow-up in two weeks. Today, she is back for it and dipstick urine shows 2+ blood and no proteinuria. You arrange for a formal urinalysis which comes back negative for glomerular red blood cells and casts. Which one of the following is the most important next step in management?
A. Urine culture.
B. DMSA.
C. Ultrasound.
D. IVP.
E. ASOT.
A. Urine culture
Based on current guidelines by the Royal Children’s Hospital (RCH), asymptomatic isolated hematuria in a child with a febrile disease other than urinary tract infection should be followed up once the febrile disease settles. Based on this guideline, this child has undergone another urine exam in 2 weeks which still shows isolated hematuria of non- glomerular origin.
For this child, a urine culture/sensitivity to exclude urinary tract infection is the most appropriate next step in management.
OPTION B : DMSA scan (DMSA, or dimercaptosuccinic acid, bound to technetium 99m) isotope is circulated through the body. 95% of the radiopharmaceutical is bound to the renal cortex and the remaining 5% is usually excreted into the urine. Modern gamma cameras capable of performing single-photon emission tomography (SPECT), are able to reconstruct 3-dimensional images of the kidneys. In children, the accepted indications for a DMSA scan include renal scars as a result of an insult (usually recurrent urinary tract infections) or reflux-associated nephropathy.
OPTION C : Ultrasound has a place in the diagnosis of renal morphology and assessing renal damage, especially in the setting of hematuria, edema, and hypertension where underlying nephritis is suspected.
OPTION D : IVP (Intravenous Pyelography) is used to check urine flow if there is concern regarding kidney, ureter, or bladder blockage, or to check the function and appearance of the urinary tract after surgery to prevent blockages from causing permanent damage to the kidneys.
OPTION E : ASOT (Anti-streptolysin O Titer) is used in the case of post-streptococcal GN in a school-aged child with macroscopic hematuria, edema due to fluid retention, and possibly a headache due to hypertension. Investigation shows elevation of urea and creatinine, normocytic anemia, elevated streptococcal markers (ASOT/anti-DNase B), and a depressed complement (C3) level.
TOPIC REVIEW
Pediatric hematuria can be divided into the following categories :
- Microscopic hematuria - (>10 RBC/microlitre) can only be detected by urinalysis as urine color remains normal
- Macroscopic hematuria - visible blood in urine without microscopy, and is more likely to come from the bladder or urethra rather than the kidney
- Persistent microscopic hematuria – three positive separate samples for blood, each taken at least one week apart, without prior exercise nor during menstruation
- Isolated asymptomatic microscopic hematuria - the presence of microscopic hematuria without clinical symptoms or any other abnormalities in the urine such as proteinuria
Assessment of pediatric hematuria should include :
* Previous history of hematuria
* Symptoms of urinary infection such as dysuria, frequency, pain, fever
* Systemic symptoms such as fatigue, edema, rash, arthralgia, or coryza
* Recent surgery or trauma including non-accidental injury
* Family history of hematuria
* History of underlying bleeding disorder or immunodeficiency
* Medication history
* Food intake (beetroot and berries can color urine to pink or red)
* Recent exercise, especially if strenuous
Causes of hematurai in the pediatric pupulation :
A . Common
-Urinary tract infection
-Perineal irritation
-Trauma
-Meatal stenosis with ulceration
B. Uncommon
-Glomerulonephritis
-Renal calculi
-Coagulation abnormalities
-Tumors
Investigation of macroscopic hematuria
* Urine Dipstick urinalysis
* Mid-stream urine Microscopy, culture, and sensitivity (MSU MCS)
* Calcium/creatinine ratio
* Blood Electrolytes including creatinine, full blood count, Antistreptiterin titer (ASOT)/anti-DNase B Complement (C3)
* Renal ultrasound
- RCH - Haematuria